7
$\begingroup$

Suppose that $r$ is an irrational number with fractional part between $1/3$ and $2/3$. Let $D_n$ be the number of distinct $n$th differences of the sequence $(\lfloor{kr}\rfloor)$. It appears that

$$D_n=(2,3,3,5,4,7,5,9,6,11,7,13,8,\ldots),$$

essentially A029579. Can someone verify that what appears here is actually true?

Example: for $r=(1+\sqrt{5})/2$, we find

$$(\lfloor{kr}\rfloor)=(1,3,4,6,8,9,11,12,14,16,17,\ldots) = A000201 = \text{ lower Wythoff sequence,}$$

\begin{align*} \Delta^1 =&(2,1,2,2,1,2,1,2,2,1,2,2,1,\ldots), & D_1=2, \\ \Delta^2 =&(1,-1,1,0,-1,1,-1,1,0,-1,1,\ldots), & D_2=3, \\ \Delta^3 =&(-2,2,-1,-1,2,-2,2,-1,-1,2,\ldots), & D_2=3. \end{align*}

$\endgroup$
1
  • 1
    $\begingroup$ I wonder if further work will show how the hypothesis that $1/3 < r < 2/3$ fits in. My experiments suggest that A029579 accounts for $D_n$ as long as $r$ is in an interval $$(\frac{1}{2n+1},\frac{1}{2n}) \text{ or } (\frac{2n-1}{2n},\frac{2n}{2n+1}).$$ The results for other intervals appear to match various modifications of A029579 that involve the numbers $8+6k$ for $k \geq 0$. $\endgroup$ May 18, 2019 at 14:40

1 Answer 1

3
$\begingroup$

It's easy to see that $D_n\leq \texttt{A029579}(n)$. Indeed, $\Delta^1$ is a Sturmian word, which is known to have exactly $n+1$ factors of length $n$.

Now, $\Delta^n$ is formed by values of the $(n-1)$-th difference operator on the factors of $\Delta^1$ of length $n$, i.e., $$\Delta^n = \left(\sum_{i=0}^{n-1} \binom{n-1}{i} (-1)^{n-1-i} \Delta^1_{k+i}\quad \big|\quad k=1,2,\dots\right).$$

For even $n$, we immediately have $D_n\leq n+1 = \texttt{A029579}(n)$.

For odd $n$, we additionally notice (I did not verify this carefully) that (i) the reverse of a Sturmian factor is a factor itself, (ii) values of the operator on a factor and its reverse are the same, and (iii) there are exactly two symmetric factors. Hence, here we have $$D_n\leq \frac{n+1-2}2 + 2 = \frac{n+3}2 = \texttt{A029579}(n).$$

It remains to prove that, besides the aforementioned cases, the operator values on factors of length $n$ are distinct.

$\endgroup$
0

Your Answer

By clicking “Post Your Answer”, you agree to our terms of service and acknowledge that you have read and understand our privacy policy and code of conduct.

Not the answer you're looking for? Browse other questions tagged or ask your own question.